LSAT and Law School Admissions Forum

Get expert LSAT preparation and law school admissions advice from PowerScore Test Preparation.

User avatar
 Dave Killoran
PowerScore Staff
  • PowerScore Staff
  • Posts: 5850
  • Joined: Mar 25, 2011
|
#26443
Complete Question Explanation
(The complete setup for this game can be found here: lsat/viewtopic.php?t=8639)


The correct answer choice is (A)

When examining the linear portion of the setup, take special note of the dual-options. A favorite trick of the test makers is to “take away” one of the variables in a dual-option to see if you recognize that the other variable is then forced into a position. Because either J or L must be second, and according to the question stem J is more popular than L, L cannot be second and J must be second. Answer choice (A) reflects that fact and is correct.
 David44357
  • Posts: 4
  • Joined: Jan 04, 2016
|
#21488
I am on page 1-26 of the PowerScore Test Preparation book. Using the local rules, I have the sequencing problem diagrammed as H>J>L>Q>S>P and L>V. From what I can tell, the first three positions should be HJL, whereas the position of the remaining variables is unknown, since V could be anywhere after L, pushing QSP into different positions. Therefore option (a) (J in the second position) and (c) (L in third) should both be right, but the answer is actually A. Was hoping someone could help me understand why (c) is not correct.
 Robert Carroll
PowerScore Staff
  • PowerScore Staff
  • Posts: 1783
  • Joined: Dec 06, 2013
|
#21490
David,

Check your rules about Q. Once J occurs, L or Q could be next.

Robert Carroll
 mchalhoub
  • Posts: 1
  • Joined: Aug 03, 2016
|
#27637
With this question, is the best strategy to redraw the diagram? Or is there a shortcut around redrawing the entire sketch?
 Shannon Parker
PowerScore Staff
  • PowerScore Staff
  • Posts: 147
  • Joined: Jun 08, 2016
|
#27656
Hey there,

Unfortunately there is no shortcut to making a diagram of the rules that are local to each question.

~Shannon
 kmaragh
  • Posts: 8
  • Joined: Jun 17, 2020
|
#76306
I understand that J must be second but couldn't we argue that Q could also be third since it according to the rules Q follows J. Also P could be 7th. So how do I pick that J being second must be true when it could be true that Q must be third or P must be seventh? Are we to assume that even though J is more popular than L, that it could be possible that it could be possible that L is before Q? But still I'm confused as to why P is seventh must be true if S is more popular than P.
 Christen Hammock
PowerScore Staff
  • PowerScore Staff
  • Posts: 61
  • Joined: May 14, 2020
|
#76329
Hi Kmaragh!

You're confusing what must be true and what could be true. You're right that Q could be third, but L could be third as well! The only one of these that absolutely must be true in every scenario is J=2.
 alexakinosh@gmail.com
  • Posts: 1
  • Joined: Aug 05, 2021
|
#89788
Hi, I know you kind of touched upon this a bit earlier but can't "P is seventh" also be true? Since S is more popular than P, doesn't P also take last place, in equal correctness to J being second?
User avatar
 Bob O'Halloran
PowerScore Staff
  • PowerScore Staff
  • Posts: 61
  • Joined: Jul 06, 2021
|
#89820
Hi Alexa,
Thank you for your question.

The key word in the question stem is Must

P could be seventh, but it could also be sixth with V being seventh. In the case of J, it has to be second, we have no other choice available.

Let us know if you have any additional questions.
 rcthomas23
  • Posts: 4
  • Joined: Nov 23, 2021
|
#93608
Hi there,

I represented the local diagram for this question with L-V below J to indicate that L came after J, like this:

H-J-Q-S-P

L-V

Would that be a correct representation? I ended up getting the right answer.

Get the most out of your LSAT Prep Plus subscription.

Analyze and track your performance with our Testing and Analytics Package.